Problem with the change of variable theorem of integrals

Click For Summary
The discussion revolves around a problem with the change of variable theorem in integrals, specifically regarding the integral ∫ from 0 to 1 of x√(1-x²). The user attempted to solve the integral using two different intervals for the variable transformation, both yielding contradictory results in sign. The first approach, using g(x) = cos(x) over [0, π/2], resulted in a positive value of 1/3, while the second approach over [-π/2, 0] led to a negative value of -1/3. The confusion arose from not accounting for the absolute value of sin(x) in the second interval, which was clarified by another participant. The user acknowledged the oversight and expressed gratitude for the correction.
IvanT
Messages
6
Reaction score
0

Homework Statement


So I came across the integral \int^{1}_{0}x\sqrt{1-x^2} and I tried to solve it in two ways using the change of variables theorem for integration, however both ways are supposed to give me the same result, but they differ in the sign and I cannot find what I am doing wrong.

Homework Equations


The change of variable theorem states the following: Let I=[a,b]. Let g: I->R be a function of class C^1, with g'(x) different from 0 for x in (a,b). Then the set g(I) is a closed interval J with end points g(a) and g(b) (not necessarily in that order). If f: J->R is continous, then \int^{g(b)}_{g(a)}f=\int^{b}_{a}(f°g)g'

The Attempt at a Solution


So, for the first solution I used g(x)=cos(x) in the interval I=[0,Pi/2]. Clearly cos(I)=[0,1] and g'(x)=-sin(x) is not 0 for any x in (0,pi/2). Since f(x)=x*sqrt(1-x^2) is continuous in [0,1] the hypotheses of the theorem are satisfied and we have \int^{g(pi/2)}_{g(0)}f=\int^{0}_{1}x\sqrt{1-x^2}=-\int^{1}_{0}x\sqrt{1-x^2}=\int^{pi/2}_{0}cos(x)sin(x)(-sin(x))=-sin^3(pi/2)/3+sin^3(0)/3=-1/3 so that \int^{1}_{0}f=1/3.
On the other hand, if I consider the same g(x)=cos(x) but now in the interval I=[-pi/2,0], we have again that cos(I)=[0,1] and the hypotheses are satisfied. However we get \int^{g(0)}_{g(-pi/2)}f=\int^{1}_{0}x\sqrt{1-x^2}=\int^{0}_{-pi/2}cos(x)sin(x)(-sin(x))=-sin^3(0)/3+sin^3(-pi/2)/3=-1/3 so that \int^{1}_{0}f=-1/3!.
I checked every step a few times already and I cannot find the mistake. Any help would be appreciated.
 
Last edited:
Physics news on Phys.org
IvanT said:

Homework Statement


So I came across the integral \int^{1}_{0}x\sqrt{1-x^2} and I tried to solve it in two ways using the change of variables theorem for integration, however both ways are supposed to give me the same result, but they differ in the sign and I cannot find what I am doing wrong.


Homework Equations


The change of variable theorem states the following: Let I=[a,b]. Let g: I->R be a function of class C^1, with g'(x) different from 0 for x in (a,b). Then the set g(I) is a closed interval J with end points g(a) and g(b) (not necessarily in that order). If f: J->R is continous, then \int^{g(b)}_{g(a)}f=\int^{b}_{a}(f°g)g'


The Attempt at a Solution


So, for the first solution I used g(x)=cos(x) in the interval I=[0,Pi/2]. Clearly cos(I)=[0,1] and g'(x)=-sin(x) is not 0 for any x in (0,pi/2). Since f(x)=x*sqrt(1-x^2) is continuous in [0,1] the hypotheses of the theorem are satisfied and we have \int^{g(pi/2)}_{g(0)}f=\int^{0}_{1}x\sqrt{1-x^2}=-\int^{1}_{0}x\sqrt{1-x^2}=\int^{pi/2}_{0}cos(x)sin(x)(-sin(x))=-sin^3(pi/2)/3+sin^3(0)/3=-1/3 so that \int^{1}_{0}f=1/3.
On the other hand, if I consider the same g(x)=cos(x) but now in the interval I=[-pi/2,0], we have again that cos(I)=[0,1] and the hypotheses are satisfied. However we get \int^{g(0)}_{g(-pi/2)}f=\int^{1}_{0}x\sqrt{1-x^2}=\int^{0}_{-pi/2}cos(x)sin(x)(-sin(x))=-sin^3(0)/3+sin^3(-pi/2)/3=-1/3 so that \int^{1}_{0}f=-1/3!.
I checked every step a few times already and I cannot find the mistake. Any help would be appreciated.

When you took √sin2(x), it's |sin(x)|, which in the second region is -sin(x)
 
algebrat said:
When you took √sin2(x), it's |sin(x)|, which in the second region is -sin(x)

Oh xD, I didn't noticed that, thanks a lot : ).
 
Question: A clock's minute hand has length 4 and its hour hand has length 3. What is the distance between the tips at the moment when it is increasing most rapidly?(Putnam Exam Question) Answer: Making assumption that both the hands moves at constant angular velocities, the answer is ## \sqrt{7} .## But don't you think this assumption is somewhat doubtful and wrong?

Similar threads

  • · Replies 5 ·
Replies
5
Views
2K
Replies
2
Views
2K
Replies
3
Views
2K
  • · Replies 3 ·
Replies
3
Views
2K
  • · Replies 11 ·
Replies
11
Views
3K
  • · Replies 105 ·
4
Replies
105
Views
6K
  • · Replies 14 ·
Replies
14
Views
2K
Replies
3
Views
2K
  • · Replies 19 ·
Replies
19
Views
2K
  • · Replies 15 ·
Replies
15
Views
2K